LSAT and Law School Admissions Forum

Get expert LSAT preparation and law school admissions advice from PowerScore Test Preparation.

 Francis O'Rourke
PowerScore Staff
  • PowerScore Staff
  • Posts: 471
  • Joined: Mar 10, 2017
|
#48771
Hi Cowboys1118!

You are completely correct that we can manage to order S, Y, and Q in spots one, two, and three, respectively, without violating directly violating any rules. Our diagram should look something like this:

N W N __ __ __ __
S Y Q __ __ __ __
1 2 3

The difficulty comes in when we think about what happens in the remaining four spaces. Since we just placed S, Y, and Q we are left with R, which is a nutrition article, and G, H, and J which are all Finance articles.

This means that in spaces four to seven we need to fit three finance articles. Since the first rule told us that we cannot place any of them consecutively, we will end up violating some rule.

For example, we can place a finance article in space four and R, a Nutrition article, in space five. Our game board would look something like this:

N W N F N __ __
S Y Q J R __ __
1 2 3 4 5

Now we have two finance articles left for the last two spaces. This is not allowed. Furthermore, doing this would end up violating the rule that states that G must appear earlier than R.

To recap, placing S, Y, and Q in the first three spaces would not directly violate any rule in those first three spaces. Doing so would force the remaining four spaces to violate multiple rules. Remember to think about how one variable could impact the entire diagram.

Let us know if this helps! :)
 gcs4v333
  • Posts: 19
  • Joined: Oct 09, 2018
|
#60279
Robert Carroll wrote:reop,

Sufficient conditions don't include necessity anyway, so "can be" is not relevant - only a necessary condition would use language like "must be." So the rule is saying:

S > Q :arrow: Q3

Contrapositive is:

Q not 3 :arrow: Q > S

Robert Carroll
I'm very confused by this, and from what I can tell this hasn't been addressed except for this one comment. Can someone please tease out why this is the case?

To circle back a bit, the second rules says: "S can be earlier than Q, only if Q is third."

The "can" really threw me for a loop. I diagrammed this rule as:

(S :longline: Q) or (Q :longline: S) :arrow: Q3

with its contrapositive:

Q3 :arrow: Q :longline: S :longline: Y


But now I'm seeing that it should simply have been:

S :longline: Q :arrow: Q3

along with its contrapositive. Why can we just disregard the "can?" It seems like that's a pretty important word.
 Robert Carroll
PowerScore Staff
  • PowerScore Staff
  • Posts: 1787
  • Joined: Dec 06, 2013
|
#61741
gcs,

We already know that S can be earlier than Q, unless something prevents it from happening. So S can be earlier than Q already. What does the statement in the rule change about that? It says that that general permissibility of S being earlier than Q has a restriction - it can happen only if something else also happens. The "can" is not a statement of possibility. That possibility is already cooked into the game. It's not affirming anything. The rule is exactly equivalent in English to "S is earlier than Q only if Q is third."

Robert Carroll
 lsatbossintraining
  • Posts: 27
  • Joined: Oct 21, 2019
|
#71753
I understand how you arrived at the “Y can’t be in slot 2” inference, but only after testing Y in two and discovering that J-G-R and H couldn’t go in slots 4-7 because of rule #1.

Just wondering if there’s a faster way to register the “Y not-law”. Not seeing how we get there just because of the third rule as noted in your explanation.

Thanks very much.
 Adam Tyson
PowerScore Staff
  • PowerScore Staff
  • Posts: 5153
  • Joined: Apr 14, 2011
|
#71838
I'm going to say no, boss-in-training - I don't think there is anything fast or obvious about that inference, and many students (including some serious LSAT bosses) that would likely miss it in the initial setup. And that's okay! There are a lot of games with subtle inferences that would be helpful if we made them up front, but which we will do just fine without discovering until we need them. While we need to invest time in our diagrams and at least try to discover some inferences, we usually cannot afford to run them all down up front. If we need them, we'll find them later through testing answer choices (after sorting losers and contenders, of course), and if we don't need them then we'll be glad we didn't waste time on them!
 lsatbossintraining
  • Posts: 27
  • Joined: Oct 21, 2019
|
#71877
Thanks Adam!
 Emily12
  • Posts: 1
  • Joined: Apr 22, 2020
|
#75041
Hello,
I really struggled with this game and missed many of the not-laws - do you have others that you think are similar to this game that you would recommend studying, since this is clearly a v weak area for me? Simply looking at other advanced linear balanced games hasn't proved that helpful, since they weren't as challenging as this one.
Thanks very much.
User avatar
 KelseyWoods
PowerScore Staff
  • PowerScore Staff
  • Posts: 1079
  • Joined: Jun 26, 2013
|
#75080
Hi Emily!

If you want to test yourself out on some difficult games, I recommend checking out our list of "Killer Games" here: https://www.powerscore.com/lsat/help/lg ... -games.cfm

There aren't a lot of Advanced Linear, Balanced games on the list because, as you've seen, usually those games are not as challenging. But you should definitely check out PT2 October 1991 Game #4: Dog Show if you haven't already! I'd also recommend checking out some of the difficult Unbalanced Advanced Linear games since they typically also rely on a lot of Not Law inferences. But challenging yourself with any of these games can help you learn about and practice making tricky inferences. That said, remember that these games are not typical and getting the basic game strategies and skills down is most important.

Hope this helps!

Best,
Kelsey
User avatar
 gingerale
  • Posts: 25
  • Joined: Feb 15, 2021
|
#84429
Hello! Could someone explain how we're supposed to come up with the final two not laws for Q and S? After testing them out on a diagram I understand why they can't work, but is there a more simple way to deduce them?
User avatar
 gingerale
  • Posts: 25
  • Joined: Feb 15, 2021
|
#84432
I understand why Y can't be 1st or 3rd, but why can't it be 2nd?

Get the most out of your LSAT Prep Plus subscription.

Analyze and track your performance with our Testing and Analytics Package.